Difference between revisions of "2018 AMC 10A Problems/Problem 7"

(Solution 3 (Brute Force))
(Solution 3 (Brute Force))
Line 27: Line 27:
  
 
==Solution 3 (Brute Force)==
 
==Solution 3 (Brute Force)==
The possible numbers for <math>n</math> are: <math>-5, -4, -3, -2, -1, 0, 1, 2, 3</math>. In total there are <math>9</math> possible values for <math>n</math>. Hence answer is <math>\boxed{\textbf{(E) }9}</math>.
+
The possible numbers for <math>n</math> are: <math>-5, -4, -3, -2, -1, 0, 1, 2, 3</math>. In total, there are <math>9</math> possible values for <math>n</math>. Hence answer is <math>\boxed{\textbf{(E) }9}</math>.
  
 
~ Little
 
~ Little

Revision as of 11:25, 29 October 2021

The following problem is from both the 2018 AMC 12A #7 and 2018 AMC 10A #7, so both problems redirect to this page.

Problem

For how many (not necessarily positive) integer values of $n$ is the value of $4000\cdot \left(\tfrac{2}{5}\right)^n$ an integer?

$\textbf{(A) }3 \qquad \textbf{(B) }4 \qquad \textbf{(C) }6 \qquad \textbf{(D) }8 \qquad \textbf{(E) }9 \qquad$

Solution 1 (Algebra)

Note that \[4000\cdot \left(\frac{2}{5}\right)^n=\left(2^5\cdot5^3\right)\cdot \left(\frac{2}{5}\right)^n=2^{5+n}\cdot5^{3-n}.\] Since this expression is an integer, we need:

  1. $5+n\geq0,$ from which $n\geq-5.$
  2. $3-n\geq0,$ from which $n\leq3.$

Taking the intersection gives $-5\leq n\leq3.$ So, there are $3-(-5)+1=\boxed{\textbf{(E) }9}$ integer values of $n.$

~MRENTHUSIASM

Solution 2 (Arithmetic)

The prime factorization of $4000$ is $2^{5}\cdot5^{3}$. Therefore, the maximum integer value for $n$ is $3$, and the minimum integer value for $n$ is $-5$. Then we must find the range from $-5$ to $3$, which is $3-(-5) + 1 = 8 + 1 = \boxed{\textbf{(E) }9}$.

Solution 3 (Brute Force)

The possible numbers for $n$ are: $-5, -4, -3, -2, -1, 0, 1, 2, 3$. In total, there are $9$ possible values for $n$. Hence answer is $\boxed{\textbf{(E) }9}$.

~ Little

Video Solutions

https://youtu.be/ZiZVIMmo260

https://youtu.be/2vz_CnxsGMA

~savannahsolver

https://youtu.be/vzyRAnpnJes

Education, the Study of Everything

https://youtu.be/ZhAZ1oPe5Ds?t=1763

~ pi_is_3.14

See Also

2018 AMC 10A (ProblemsAnswer KeyResources)
Preceded by
Problem 6
Followed by
Problem 8
1 2 3 4 5 6 7 8 9 10 11 12 13 14 15 16 17 18 19 20 21 22 23 24 25
All AMC 10 Problems and Solutions
2018 AMC 12A (ProblemsAnswer KeyResources)
Preceded by
Problem 6
Followed by
Problem 8
1 2 3 4 5 6 7 8 9 10 11 12 13 14 15 16 17 18 19 20 21 22 23 24 25
All AMC 12 Problems and Solutions

The problems on this page are copyrighted by the Mathematical Association of America's American Mathematics Competitions. AMC logo.png